Last visit was: 24 Apr 2024, 16:46 It is currently 24 Apr 2024, 16:46

Close
GMAT Club Daily Prep
Thank you for using the timer - this advanced tool can estimate your performance and suggest more practice questions. We have subscribed you to Daily Prep Questions via email.

Customized
for You

we will pick new questions that match your level based on your Timer History

Track
Your Progress

every week, we’ll send you an estimated GMAT score based on your performance

Practice
Pays

we will pick new questions that match your level based on your Timer History
Not interested in getting valuable practice questions and articles delivered to your email? No problem, unsubscribe here.
Close
Request Expert Reply
Confirm Cancel
SORT BY:
Date
Tags:
Show Tags
Hide Tags
User avatar
Manager
Manager
Joined: 13 May 2010
Posts: 81
Own Kudos [?]: 761 [66]
Given Kudos: 7
Send PM
Most Helpful Reply
User avatar
Manhattan Prep Instructor
Joined: 21 Jan 2010
Affiliations: ManhattanGMAT
Posts: 323
Own Kudos [?]: 7018 [12]
Given Kudos: 11
Location: San Francisco
Concentration: Journalism
 Q47  V47 GMAT 2: 770  Q49  V48
Send PM
User avatar
BSchool Moderator
Joined: 19 Feb 2010
Posts: 271
Own Kudos [?]: 521 [9]
Given Kudos: 76
Send PM
General Discussion
User avatar
Intern
Intern
Joined: 02 May 2010
Posts: 31
Own Kudos [?]: 165 [2]
Given Kudos: 3
Schools:IU, UT Dallas, Univ of Georgia, Univ of Arkansas, Miami University
WE 1: 5.5 Yrs IT
Send PM
Re: The workers at Bell Manufacturing will shortly go on strike unless the [#permalink]
2
Kudos
Nice Question gmatcracker2010. IMO C and OA is C too.

The question asks to identify the assumption made for concluding that the subsidiaries will be sold. If some of the subsidiaries are sold, then the increase in wages is certain. Increase in wages will in turn imply that the workers will not go on a strike.

You can also insert all the assumption choices in the argument and boil down to the one that makes sense logically. I have inserted option C to show that it makes sense.

The workers at Bell Manufacturing will shortly go on strike unless the management increases their wages. As Bell’s president is well aware, however, in order to increase the worker’s wages, Bell would have to sell off some of its subsidiaries. Since, the workers at Bell Manufacturing will not be going on strike, some of Bell’s subsidiaries will be sold.

(A) Bell Manufacturing will begin to suffer increased losses.
(B) Bell’s management will refuse to increase its worker’s wages.
(C) The workers at Bell Manufacturing will not be going on strike.
(D) Bell’s president has the authority to offer the workers their desired wage increase.
(E) Bell’s workers will not accept a package of improved benefits in place of their desired wage increase.

Originally posted by sjayasa on 10 Jun 2010, 00:27.
Last edited by sjayasa on 10 Jun 2010, 05:44, edited 1 time in total.
Intern
Intern
Joined: 18 Apr 2016
Posts: 5
Own Kudos [?]: 4 [0]
Given Kudos: 16
Send PM
Re: The workers at Bell Manufacturing will shortly go on strike unless the [#permalink]
The workers at Bell Manufacturing will shortly go on strike unless the management increases their wages. As Bell’s president is well aware, however, in order to increase the worker’s wages, Bell would have to sell off some of its subsidiaries. So, some of Bell’s subsidiaries will be sold.

The conclusion above is properly drawn if which one of the following is assumed?

(A) Bell Manufacturing will begin to suffer increased losses.
(B) Bell’s management will refuse to increase its worker’s wages.
(C) The workers at Bell Manufacturing will not be going on strike.
(D) Bell’s president has the authority to offer the workers their desired wage increase.
(E) Bell’s workers will not accept a package of improved benefits in place of their desired wage increase.

For the question workers at Bell Manufacturing:ths gist of this arguments is that Bell industries is about to have a strike so-->sell subsidiaries -->increase worker wages


(A) Bell Manufacturing will begin to suffer increased losses.
Talking about the future-Out of scope for conclusion questions
(B) Bell’s management will refuse to increase its worker’s wages.
If this is true the argument collapses
(C) The workers at Bell Manufacturing will not be going on strike.
One simple understanding is that it will increase the wage by selling the subsidiaries.Option C is the only option within the scope
(D) Bell’s president has the authority to offer the workers their desired wage increase.
We are not provided any information that whether president has the authority or not.So out of scope
(E) Bell’s workers will not accept a package of improved benefits in place of their desired wage increase.
Out of scope
Intern
Intern
Joined: 18 Sep 2016
Posts: 3
Own Kudos [?]: [0]
Given Kudos: 23
Send PM
Re: The workers at Bell Manufacturing will shortly go on strike unless the [#permalink]
I choose E
because if workers will accept a package of improved benefits in place of their desired wage increase then there will be no need to sell subsidiaries. Since the conclusion says that they will sell subsidiaries that means they assume that there is no other way ( improved benefits ) but increase wages to stop the strike
C is the obvious result after selling the subsidies I can't understand y C is right
thanks in advance
GMAT Club Verbal Expert
Joined: 13 Aug 2009
Status: GMAT/GRE/LSAT tutors
Posts: 6920
Own Kudos [?]: 63658 [1]
Given Kudos: 1773
Location: United States (CO)
GMAT 1: 780 Q51 V46
GMAT 2: 800 Q51 V51
GRE 1: Q170 V170

GRE 2: Q170 V170
Send PM
Re: The workers at Bell Manufacturing will shortly go on strike unless the [#permalink]
1
Kudos
Expert Reply
Ridhimajain96 wrote:
I choose E
because if workers will accept a package of improved benefits in place of their desired wage increase then there will be no need to sell subsidiaries. Since the conclusion says that they will sell subsidiaries that means they assume that there is no other way ( improved benefits ) but increase wages to stop the strike
C is the obvious result after selling the subsidies I can't understand y C is right
thanks in advance

Quote:
The conclusion above is properly drawn if which one of the following is assumed?

(A) Bell Manufacturing will begin to suffer increased losses.
(B) Bell’s management will refuse to increase its worker’s wages.
(C) The workers at Bell Manufacturing will not be going on strike.
(D) Bell’s president has the authority to offer the workers their desired wage increase.
(E) Bell’s workers will not accept a package of improved benefits in place of their desired wage increase.

The author correctly reasons that some of Bell's subsidiaries will have to be sold in order to avoid a strike. But we can only accurately conclude that some of the subisidiaries will be sold IF we assume that the strike is to be avoided. What if the strike is simply allowed to happen? What if the president doesn't care whether the workers go on strike? If that's the case, there would be no reason to sell the subsidiaries.

Thus, the argument relies on the assumption stated in choice (C).

As for choice (E), we are already told that workers at Bell Manufacturing will shortly go on strike unless the management increases their wages. This statement describes a NECESSARY condition. In order to avoid a strike, management MUST increase wages. Since we are given this fact, we have to assume that no other solution will prevent the strike.

Choice (E) simply restates something that is already implied by this premise. The author does not have to make this assumption since we are already given the fact that only a wage increase will prevent a strike.

I hope this helps!
Current Student
Joined: 24 Oct 2016
Posts: 166
Own Kudos [?]: 228 [0]
Given Kudos: 116
Location: India
Concentration: Technology, Strategy
GMAT 1: 710 Q49 V38
GMAT 2: 760 Q50 V44 (Online)
GPA: 3.61
Send PM
Re: The workers at Bell Manufacturing will shortly go on strike unless the [#permalink]
gmatcracker2010 wrote:
The workers at Bell Manufacturing will shortly go on strike unless the management increases their wages. As Bell’s president is well aware, however, in order to increase the worker’s wages, Bell would have to sell off some of its subsidiaries. So, some of Bell’s subsidiaries will be sold.

The conclusion above is properly drawn if which one of the following is assumed?

(A) Bell Manufacturing will begin to suffer increased losses.
(B) Bell’s management will refuse to increase its worker’s wages.
(C) The workers at Bell Manufacturing will not be going on strike.
(D) Bell’s president has the authority to offer the workers their desired wage increase.
(E) Bell’s workers will not accept a package of improved benefits in place of their desired wage increase.

Source: LSAT

OA is c


I have marked d. Can somebody please explain?


The conclusion is: some of Bell’s subsidiaries will be sold

This stimulus can be seen as a string of conditionals and we are looking for an answer that ensures the conclusion takes place.

The workers at Bell Manufacturing will shortly go on strike unless the management increases their wages[Conditional 1]

If workers do not go on strike -> management has increased wages

As Bell’s president is well aware, however, in order to increase the worker’s wages, Bell would have to sell off some of its subsidiaries.[Conditional 2]

If the management has increased wages, Bell would have sold some of its subsidiaries

Clearly, as per the chain, we need the workers to not go on strike to ensure that Bell has sold some of its subsidiaries. Hence option (C) is the assumption. Hope that helps :-D
Manager
Manager
Joined: 04 Oct 2018
Posts: 121
Own Kudos [?]: 1034 [0]
Given Kudos: 141
Location: Viet Nam
Send PM
Re: The workers at Bell Manufacturing will shortly go on strike unless the [#permalink]
gmatcracker2010 wrote:
The workers at Bell Manufacturing will shortly go on strike unless the management increases their wages. As Bell’s president is well aware, however, in order to increase the worker’s wages, Bell would have to sell off some of its subsidiaries. So, some of Bell’s subsidiaries will be sold.

The conclusion above is properly drawn if which one of the following is assumed?

(A) Bell Manufacturing will begin to suffer increased losses.
(B) Bell’s management will refuse to increase its worker’s wages.
(C) The workers at Bell Manufacturing will not be going on strike.
(D) Bell’s president has the authority to offer the workers their desired wage increase.
(E) Bell’s workers will not accept a package of improved benefits in place of their desired wage increase.

Source: LSAT

OA is c

I have marked d. Can somebody please explain?


Conclusion: some of Bell’s subsidiaries will be sold. => Wage is increased. The premise points out that if the wage is increased, the workers would stop going on strike=> C matches.
Senior Manager
Senior Manager
Joined: 10 Apr 2018
Posts: 342
Own Kudos [?]: 200 [0]
Given Kudos: 217
Concentration: Leadership, Strategy
GMAT 1: 600 Q44 V28
GPA: 3.56
WE:Engineering (Computer Software)
Send PM
Re: The workers at Bell Manufacturing will shortly go on strike unless the [#permalink]
Ridhimajain96 wrote:
I choose E
because if workers will accept a package of improved benefits in place of their desired wage increase then there will be no need to sell subsidiaries. Since the conclusion says that they will sell subsidiaries that means they assume that there is no other way ( improved benefits ) but increase wages to stop the strike
C is the obvious result after selling the subsidies I can't understand y C is right
thanks in advance


Option 'C' had tempted me but I ended choosing E.
gmatwhiz,
Please explain this using your framework method. In fact, I choose E with same thought process and Ridhimajain96 has similar thought process.
Please help.
Intern
Intern
Joined: 06 Dec 2021
Posts: 11
Own Kudos [?]: 5 [0]
Given Kudos: 95
Concentration: Finance, International Business
GMAT 1: 580 Q45 V25
Send PM
The workers at Bell Manufacturing will shortly go on strike unless the [#permalink]
GMATNinja wrote:
Ridhimajain96 wrote:
I choose E
because if workers will accept a package of improved benefits in place of their desired wage increase then there will be no need to sell subsidiaries. Since the conclusion says that they will sell subsidiaries that means they assume that there is no other way ( improved benefits ) but increase wages to stop the strike
C is the obvious result after selling the subsidies I can't understand y C is right
thanks in advance

Quote:
The conclusion above is properly drawn if which one of the following is assumed?

(A) Bell Manufacturing will begin to suffer increased losses.
(B) Bell’s management will refuse to increase its worker’s wages.
(C) The workers at Bell Manufacturing will not be going on strike.
(D) Bell’s president has the authority to offer the workers their desired wage increase.
(E) Bell’s workers will not accept a package of improved benefits in place of their desired wage increase.

The author correctly reasons that some of Bell's subsidiaries will have to be sold in order to avoid a strike. But we can only accurately conclude that some of the subisidiaries will be sold IF we assume that the strike is to be avoided. What if the strike is simply allowed to happen? What if the president doesn't care whether the workers go on strike? If that's the case, there would be no reason to sell the subsidiaries.

Thus, the argument relies on the assumption stated in choice (C).

As for choice (E), we are already told that workers at Bell Manufacturing will shortly go on strike unless the management increases their wages. This statement describes a NECESSARY condition. In order to avoid a strike, management MUST increase wages. Since we are given this fact, we have to assume that no other solution will prevent the strike.

Choice (E) simply restates something that is already implied by this premise. The author does not have to make this assumption since we are already given the fact that only a wage increase will prevent a strike.

I hope this helps!


GMATNinja I think in the highlighted part you meant that the statement described is a SUFFICIENT condition, since a sufficient condition leaves no room for assuming other possibilities, but a necessary condition actually does it. Let me know whether I am correct or not? :)
Senior Manager
Senior Manager
Joined: 11 May 2021
Posts: 272
Own Kudos [?]: 115 [0]
Given Kudos: 446
Send PM
The workers at Bell Manufacturing will shortly go on strike unless the [#permalink]
KarishmaB - Hi Karishma, this is to better understand the necessary/sufficient condition here-

In the above question, if the management increases the wages, then there is still a possibility that the workers may / may not go on a strike. But if the management doesn't increase the wages, then 100% the workers will go on the strike right? Is my understanding correct ? Can you share more color on necessary/sufficient condition in this example.

I see a conflict in the approach to the above and the below question:
https://gmatclub.com/forum/unless-negot ... s#p3252226

There we said that if negotiations begin soon, then cease fire may or may not be violated
Similarly then in the above question, how can we say that if management increases the wages, the workers will not go on strike? They may because the management increase is a necessary but not a sufficient condition.
Tutor
Joined: 16 Oct 2010
Posts: 14817
Own Kudos [?]: 64900 [0]
Given Kudos: 426
Location: Pune, India
Send PM
Re: The workers at Bell Manufacturing will shortly go on strike unless the [#permalink]
Expert Reply
gmatcracker2010 wrote:
The workers at Bell Manufacturing will shortly go on strike unless the management increases their wages. As Bell’s president is well aware, however, in order to increase the worker’s wages, Bell would have to sell off some of its subsidiaries. So, some of Bell’s subsidiaries will be sold.

The conclusion above is properly drawn if which one of the following is assumed?

(A) Bell Manufacturing will begin to suffer increased losses.
(B) Bell’s management will refuse to increase its worker’s wages.
(C) The workers at Bell Manufacturing will not be going on strike.
(D) Bell’s president has the authority to offer the workers their desired wage increase.
(E) Bell’s workers will not accept a package of improved benefits in place of their desired wage increase.


Keep in Mind - The 'unless clause' is a necessary condition for negated main clause.

So 'Unless A, B' means 'Only if A, then not B'

The workers at Bell Manufacturing will shortly go on strike unless the management increases their wages.

Highlighted is the unless clause. So 'wage increase' is NECESSARY for 'NOT going on strike.'

Then here are the premises:
'Wage increase' is NECESSARY for 'NOT going on strike.'
Selling subsidiaries is necessary in order to increase wages.

Conclusion: Some of Bell’s subsidiaries will be sold.

To conclude that some subsidiaries will be sold, we are assuming that workers will not go on strike. For not going on strike, it is necessary that wage will increase. For wage increase, it is necessary to sell subsidiaries.
and hence we are concluding that subsidiaries will be sold. Makes sense. We are "properly drawing" the conclusion. Note that the question is a "sufficient assumption" question.


Answer (C)

Check this video for necessary and sufficient conditions: https://youtu.be/MmlwcTlHZz8
Tutor
Joined: 16 Oct 2010
Posts: 14817
Own Kudos [?]: 64900 [1]
Given Kudos: 426
Location: Pune, India
Send PM
Re: The workers at Bell Manufacturing will shortly go on strike unless the [#permalink]
1
Kudos
Expert Reply
kittle wrote:
KarishmaB - Hi Karishma, this is to better understand the necessary/sufficient condition here-

In the above question, if the management increases the wages, then there is still a possibility that the workers may / may not go on a strike. But if the management doesn't increase the wages, then 100% the workers will go on the strike right? Is my understanding correct ? Can you share more color on necessary/sufficient condition in this example.

I see a conflict in the approach to the above and the below question:
https://gmatclub.com/forum/unless-negot ... s#p3252226

There we said that if negotiations begin soon, then cease fire may or may not be violated
Similarly then in the above question, how can we say that if management increases the wages, the workers will not go on strike? They may because the management increase is a necessary but not a sufficient condition.


As for the link of the other question you mentioned, note that the question stems are different.

In our current question we need to properly draw the conclusion i.e. the conclusion must follow from the premises and the assumption together. So to be sure that some subsidies will be sold, we need to say that strike will not happen because wage increase is necessary for strike not happening and selling subsidies is necessary for wage increase.
Then, if we know that strike did not happen, we know for sure that wage increase must have happened and subsidies must have been sold. Our conclusion follows.

In the other question, we are asked what must be true and what is not necessary to be true.
We are given that negotiations did begin soon (the necessary condition is true). Does this mean that the cease fire will not be violated? No. Negotiations are necessary, not sufficient for cease fire to not be violated.
GMAT Club Verbal Expert
Joined: 13 Aug 2009
Status: GMAT/GRE/LSAT tutors
Posts: 6920
Own Kudos [?]: 63658 [1]
Given Kudos: 1773
Location: United States (CO)
GMAT 1: 780 Q51 V46
GMAT 2: 800 Q51 V51
GRE 1: Q170 V170

GRE 2: Q170 V170
Send PM
Re: The workers at Bell Manufacturing will shortly go on strike unless the [#permalink]
1
Kudos
Expert Reply
Crismore wrote:
GMATNinja wrote:
Ridhimajain96 wrote:
I choose E

because if workers will accept a package of improved benefits in place of their desired wage increase then there will be no need to sell subsidiaries. Since the conclusion says that they will sell subsidiaries that means they assume that there is no other way ( improved benefits ) but increase wages to stop the strike

C is the obvious result after selling the subsidies I can't understand y C is right

thanks in advance


Quote:
The conclusion above is properly drawn if which one of the following is assumed?

(A) Bell Manufacturing will begin to suffer increased losses.

(B) Bell’s management will refuse to increase its worker’s wages.

(C) The workers at Bell Manufacturing will not be going on strike.

(D) Bell’s president has the authority to offer the workers their desired wage increase.

(E) Bell’s workers will not accept a package of improved benefits in place of their desired wage increase.


The author correctly reasons that some of Bell's subsidiaries will have to be sold in order to avoid a strike. But we can only accurately conclude that some of the subisidiaries will be sold IF we assume that the strike is to be avoided. What if the strike is simply allowed to happen? What if the president doesn't care whether the workers go on strike? If that's the case, there would be no reason to sell the subsidiaries.

Thus, the argument relies on the assumption stated in choice (C).

As for choice (E), we are already told that workers at Bell Manufacturing will shortly go on strike unless the management increases their wages. This statement describes a NECESSARY condition. In order to avoid a strike, management MUST increase wages. Since we are given this fact, we have to assume that no other solution will prevent the strike.

Choice (E) simply restates something that is already implied by this premise. The author does not have to make this assumption since we are already given the fact that only a wage increase will prevent a strike.

I hope this helps!


GMATNinja I think in the highlighted part you meant that the statement described is a SUFFICIENT condition, since a sufficient condition leaves no room for assuming other possibilities, but a necessary condition actually does it. Let me know whether I am correct or not? :)

Sorry that I'm late to the party here! You raise a great question -- what's the difference between a necessary condition and a sufficient one?

Well, a sufficient condition allows a conclusion to be drawn. For example, if my friend Albert lives in Iceland, is that SUFFICIENT to conclude that he lives on earth? Yes, it is. However, it isn't necessary. To live on earth, it isn't absolutely NECESSARY that you live in Iceland.

Let's take that the other way. If my friend lives in Iceland, then living on earth is a NECESSARY condition for that to be true. But it isn't SUFFICIENT. In other words, just because Albert lives on earth doesn't mean he lives in Iceland.

Let's apply that logic to answer choice (E):

Quote:
Bell’s workers will not accept a package of improved benefits in place of their desired wage increase.

Notice this isn't SUFFICIENT to reach the conclusion. Just because workers "will not accept a package of improved benefits in place of a wage increase" doesn't mean some of Bell's subsidiaries will be sold. Even if the workers don't accept the package, we don't have to conclude that some of Bell's subsidiaries will be sold.

Put another way -- even if (E) is true, other outcomes besides Bell selling subsidiaries are possible. Maybe the president will simply allow the workers to go on strike, in which case Bell wouldn't have to raise wages or sell subsidiaries?

Bottom line, since (E) doesn't guarantee the conclusion, it isn't sufficient.

However, (E) is necessary. We need to assume that Bell's workers will NOT accept the package. Because if they did, the strike could be avoided without raising wages or selling subsidiaries. Since we're looking for a SUFFICIENT condition, however, not a NECESSARY one, (E) is incorrect.

I hope that helps!
Intern
Intern
Joined: 15 Oct 2022
Posts: 7
Own Kudos [?]: 2 [0]
Given Kudos: 128
Send PM
Re: The workers at Bell Manufacturing will shortly go on strike unless the [#permalink]
not an 800+ question honestly. I think it's best to reclassify
Math Expert
Joined: 02 Sep 2009
Posts: 92900
Own Kudos [?]: 618811 [0]
Given Kudos: 81588
Send PM
Re: The workers at Bell Manufacturing will shortly go on strike unless the [#permalink]
Expert Reply
Jammy88 wrote:
not an 800+ question honestly. I think it's best to reclassify


We do not assign the difficulty level manually. The difficulty level of a question on the site is determined automatically based on various parameters collected from users' attempts, such as the percentage of correct answers and the time taken to answer the question. You can find the difficulty level of a question and its related statistics in the first post. So, the difficulty level of this question is 800+ Level based on the timer attempts from the users.
GMAT Club Bot
Re: The workers at Bell Manufacturing will shortly go on strike unless the [#permalink]
Moderators:
GMAT Club Verbal Expert
6920 posts
GMAT Club Verbal Expert
238 posts
CR Forum Moderator
832 posts

Powered by phpBB © phpBB Group | Emoji artwork provided by EmojiOne